Convergence of $sigma-$algebra for converging stopping time












2












$begingroup$


Given a filtration, ${mathcal{F}_t},tin[0,infty).$ Let $T_n$ be a sequence of stopping time that converges to $T$ and $T_nle T_{n+1}.$ We have correpsonding $sigma-$algebra, ${mathcal{F}_{T_n}}$ and $mathcal{F}_T.$



Now, denote $mathcal{F}'=sigma(mathcal{F}_{T_n}:n=1,2,cdots),$ i.e., the $sigma-$algebra generated by all $mathcal{F}_{T_n}$.



Q: Will $mathcal{F}'=mathcal{F}_T$ holds?



I believe the condition that the filtration is left continuous is needed, since one can take $T_n,T$ to be constant. Let's assume that.





My try:




  • That $mathcal{F}'subsetmathcal{F}_T$ is trivial, since all $mathcal{F}_{T_n}subsetmathcal{F}_T$ by $T_nle T$. We left to show $mathcal{F}_Tsubsetmathcal{F}'.$

  • By definition, $Ainmathcal{F}_T$ is equivalent to $Acap{Tle t}inmathcal{F}_t$ for any $t$. How can one deduce from here that $Ain mathcal{F}'.$ Got stuck here.

  • In Approximation of a unbounded stopping time and convergence of respective $sigma$-algebras, saz gives an approach for discrete time. For $Ainmathcal{F}_T, A$ can be decomposed as,
    $$A = cup_{n=1}^{infty} (Acap{Tle n})cup(Acap{T=infty}) = cup_{n=1}^{infty} A_ncup A_infty.$$
    Then show $A_ninmathcal{F}_{T_n}.$ We have $A_ninmathcal{F}_n, $ we require to show $A_ncap{T_nle t}inmathcal{F_t},$ this is so if $tge n$ since ${T_nle t}inmathcal{F}_t$. But how about $t<n?$ I was stuck here.




Update:
One can define, $mathcal{F}_{S-}=$ the $sigma-$algebra generated by $mathcal{F}_{0+}=cap_{s>0}mathcal{F}_s$ and the sets ${S>t}capmathcal{F_t}.$ then when $S=s$ is a constant, $mathcal{F}_{S-}=sigma(mathcal{F}_u:u<s)=mathcal{F}_{s-}.$ So this is simply the generalization of left limit. Then one can prove that,
$$mathcal{F}_{T-}=mathcal{F}'.$$



So now, the question may become to show,
$$ mathcal{F}_{T-}=mathcal{F}_{T} .$$





Any hint is appreciated!










share|cite|improve this question











$endgroup$

















    2












    $begingroup$


    Given a filtration, ${mathcal{F}_t},tin[0,infty).$ Let $T_n$ be a sequence of stopping time that converges to $T$ and $T_nle T_{n+1}.$ We have correpsonding $sigma-$algebra, ${mathcal{F}_{T_n}}$ and $mathcal{F}_T.$



    Now, denote $mathcal{F}'=sigma(mathcal{F}_{T_n}:n=1,2,cdots),$ i.e., the $sigma-$algebra generated by all $mathcal{F}_{T_n}$.



    Q: Will $mathcal{F}'=mathcal{F}_T$ holds?



    I believe the condition that the filtration is left continuous is needed, since one can take $T_n,T$ to be constant. Let's assume that.





    My try:




    • That $mathcal{F}'subsetmathcal{F}_T$ is trivial, since all $mathcal{F}_{T_n}subsetmathcal{F}_T$ by $T_nle T$. We left to show $mathcal{F}_Tsubsetmathcal{F}'.$

    • By definition, $Ainmathcal{F}_T$ is equivalent to $Acap{Tle t}inmathcal{F}_t$ for any $t$. How can one deduce from here that $Ain mathcal{F}'.$ Got stuck here.

    • In Approximation of a unbounded stopping time and convergence of respective $sigma$-algebras, saz gives an approach for discrete time. For $Ainmathcal{F}_T, A$ can be decomposed as,
      $$A = cup_{n=1}^{infty} (Acap{Tle n})cup(Acap{T=infty}) = cup_{n=1}^{infty} A_ncup A_infty.$$
      Then show $A_ninmathcal{F}_{T_n}.$ We have $A_ninmathcal{F}_n, $ we require to show $A_ncap{T_nle t}inmathcal{F_t},$ this is so if $tge n$ since ${T_nle t}inmathcal{F}_t$. But how about $t<n?$ I was stuck here.




    Update:
    One can define, $mathcal{F}_{S-}=$ the $sigma-$algebra generated by $mathcal{F}_{0+}=cap_{s>0}mathcal{F}_s$ and the sets ${S>t}capmathcal{F_t}.$ then when $S=s$ is a constant, $mathcal{F}_{S-}=sigma(mathcal{F}_u:u<s)=mathcal{F}_{s-}.$ So this is simply the generalization of left limit. Then one can prove that,
    $$mathcal{F}_{T-}=mathcal{F}'.$$



    So now, the question may become to show,
    $$ mathcal{F}_{T-}=mathcal{F}_{T} .$$





    Any hint is appreciated!










    share|cite|improve this question











    $endgroup$















      2












      2








      2





      $begingroup$


      Given a filtration, ${mathcal{F}_t},tin[0,infty).$ Let $T_n$ be a sequence of stopping time that converges to $T$ and $T_nle T_{n+1}.$ We have correpsonding $sigma-$algebra, ${mathcal{F}_{T_n}}$ and $mathcal{F}_T.$



      Now, denote $mathcal{F}'=sigma(mathcal{F}_{T_n}:n=1,2,cdots),$ i.e., the $sigma-$algebra generated by all $mathcal{F}_{T_n}$.



      Q: Will $mathcal{F}'=mathcal{F}_T$ holds?



      I believe the condition that the filtration is left continuous is needed, since one can take $T_n,T$ to be constant. Let's assume that.





      My try:




      • That $mathcal{F}'subsetmathcal{F}_T$ is trivial, since all $mathcal{F}_{T_n}subsetmathcal{F}_T$ by $T_nle T$. We left to show $mathcal{F}_Tsubsetmathcal{F}'.$

      • By definition, $Ainmathcal{F}_T$ is equivalent to $Acap{Tle t}inmathcal{F}_t$ for any $t$. How can one deduce from here that $Ain mathcal{F}'.$ Got stuck here.

      • In Approximation of a unbounded stopping time and convergence of respective $sigma$-algebras, saz gives an approach for discrete time. For $Ainmathcal{F}_T, A$ can be decomposed as,
        $$A = cup_{n=1}^{infty} (Acap{Tle n})cup(Acap{T=infty}) = cup_{n=1}^{infty} A_ncup A_infty.$$
        Then show $A_ninmathcal{F}_{T_n}.$ We have $A_ninmathcal{F}_n, $ we require to show $A_ncap{T_nle t}inmathcal{F_t},$ this is so if $tge n$ since ${T_nle t}inmathcal{F}_t$. But how about $t<n?$ I was stuck here.




      Update:
      One can define, $mathcal{F}_{S-}=$ the $sigma-$algebra generated by $mathcal{F}_{0+}=cap_{s>0}mathcal{F}_s$ and the sets ${S>t}capmathcal{F_t}.$ then when $S=s$ is a constant, $mathcal{F}_{S-}=sigma(mathcal{F}_u:u<s)=mathcal{F}_{s-}.$ So this is simply the generalization of left limit. Then one can prove that,
      $$mathcal{F}_{T-}=mathcal{F}'.$$



      So now, the question may become to show,
      $$ mathcal{F}_{T-}=mathcal{F}_{T} .$$





      Any hint is appreciated!










      share|cite|improve this question











      $endgroup$




      Given a filtration, ${mathcal{F}_t},tin[0,infty).$ Let $T_n$ be a sequence of stopping time that converges to $T$ and $T_nle T_{n+1}.$ We have correpsonding $sigma-$algebra, ${mathcal{F}_{T_n}}$ and $mathcal{F}_T.$



      Now, denote $mathcal{F}'=sigma(mathcal{F}_{T_n}:n=1,2,cdots),$ i.e., the $sigma-$algebra generated by all $mathcal{F}_{T_n}$.



      Q: Will $mathcal{F}'=mathcal{F}_T$ holds?



      I believe the condition that the filtration is left continuous is needed, since one can take $T_n,T$ to be constant. Let's assume that.





      My try:




      • That $mathcal{F}'subsetmathcal{F}_T$ is trivial, since all $mathcal{F}_{T_n}subsetmathcal{F}_T$ by $T_nle T$. We left to show $mathcal{F}_Tsubsetmathcal{F}'.$

      • By definition, $Ainmathcal{F}_T$ is equivalent to $Acap{Tle t}inmathcal{F}_t$ for any $t$. How can one deduce from here that $Ain mathcal{F}'.$ Got stuck here.

      • In Approximation of a unbounded stopping time and convergence of respective $sigma$-algebras, saz gives an approach for discrete time. For $Ainmathcal{F}_T, A$ can be decomposed as,
        $$A = cup_{n=1}^{infty} (Acap{Tle n})cup(Acap{T=infty}) = cup_{n=1}^{infty} A_ncup A_infty.$$
        Then show $A_ninmathcal{F}_{T_n}.$ We have $A_ninmathcal{F}_n, $ we require to show $A_ncap{T_nle t}inmathcal{F_t},$ this is so if $tge n$ since ${T_nle t}inmathcal{F}_t$. But how about $t<n?$ I was stuck here.




      Update:
      One can define, $mathcal{F}_{S-}=$ the $sigma-$algebra generated by $mathcal{F}_{0+}=cap_{s>0}mathcal{F}_s$ and the sets ${S>t}capmathcal{F_t}.$ then when $S=s$ is a constant, $mathcal{F}_{S-}=sigma(mathcal{F}_u:u<s)=mathcal{F}_{s-}.$ So this is simply the generalization of left limit. Then one can prove that,
      $$mathcal{F}_{T-}=mathcal{F}'.$$



      So now, the question may become to show,
      $$ mathcal{F}_{T-}=mathcal{F}_{T} .$$





      Any hint is appreciated!







      probability-theory measure-theory stopping-times






      share|cite|improve this question















      share|cite|improve this question













      share|cite|improve this question




      share|cite|improve this question








      edited Jan 20 at 10:04







      user103567

















      asked Jan 20 at 9:15









      user103567user103567

      1136




      1136






















          1 Answer
          1






          active

          oldest

          votes


















          0












          $begingroup$

          The inclusion $mathcal F'subset mathcal F_T$ may be strict.



          Example: Let $(Omega,mathcal F,Bbb P)$ be a probability space on which is defined a standard normal random variable $Z$. (So that $Bbb P[Z=0]=0$.) Let $(mathcal F_t)$ be the filtration generated by the process $X$ defined by
          $$
          X_t=cases{1,&$0le t<1$,cr Z,&$tge 1$.cr}
          $$

          For $T_n$ take the constant $1-1/n$, and then $T=1$. Yiu have $mathcal F_{T_n}=mathcal F' = {emptyset,Omega}$ for all $n$, but $mathcal F_T=sigma{Z}$.
          The key here is that $T$ is predictable, and "something new" happens at time $T$.






          share|cite|improve this answer









          $endgroup$













          • $begingroup$
            I agree with you that the inclusion may be strict, but your example doesn't work because the OP is assuming that $mathcal{F}_t = mathcal{F}_{t-}$ for all $t$.
            $endgroup$
            – saz
            Jan 20 at 17:56










          • $begingroup$
            Missed that. To make an example with random $T$, start a Brownian motion at 0 and run it until the time $T$ that it first hits 1. At that time have the process jump to a random position independent of the past, with a standard normal law, and then stay there forever.
            $endgroup$
            – John Dawkins
            Jan 21 at 14:57











          Your Answer





          StackExchange.ifUsing("editor", function () {
          return StackExchange.using("mathjaxEditing", function () {
          StackExchange.MarkdownEditor.creationCallbacks.add(function (editor, postfix) {
          StackExchange.mathjaxEditing.prepareWmdForMathJax(editor, postfix, [["$", "$"], ["\\(","\\)"]]);
          });
          });
          }, "mathjax-editing");

          StackExchange.ready(function() {
          var channelOptions = {
          tags: "".split(" "),
          id: "69"
          };
          initTagRenderer("".split(" "), "".split(" "), channelOptions);

          StackExchange.using("externalEditor", function() {
          // Have to fire editor after snippets, if snippets enabled
          if (StackExchange.settings.snippets.snippetsEnabled) {
          StackExchange.using("snippets", function() {
          createEditor();
          });
          }
          else {
          createEditor();
          }
          });

          function createEditor() {
          StackExchange.prepareEditor({
          heartbeatType: 'answer',
          autoActivateHeartbeat: false,
          convertImagesToLinks: true,
          noModals: true,
          showLowRepImageUploadWarning: true,
          reputationToPostImages: 10,
          bindNavPrevention: true,
          postfix: "",
          imageUploader: {
          brandingHtml: "Powered by u003ca class="icon-imgur-white" href="https://imgur.com/"u003eu003c/au003e",
          contentPolicyHtml: "User contributions licensed under u003ca href="https://creativecommons.org/licenses/by-sa/3.0/"u003ecc by-sa 3.0 with attribution requiredu003c/au003e u003ca href="https://stackoverflow.com/legal/content-policy"u003e(content policy)u003c/au003e",
          allowUrls: true
          },
          noCode: true, onDemand: true,
          discardSelector: ".discard-answer"
          ,immediatelyShowMarkdownHelp:true
          });


          }
          });














          draft saved

          draft discarded


















          StackExchange.ready(
          function () {
          StackExchange.openid.initPostLogin('.new-post-login', 'https%3a%2f%2fmath.stackexchange.com%2fquestions%2f3080350%2fconvergence-of-sigma-algebra-for-converging-stopping-time%23new-answer', 'question_page');
          }
          );

          Post as a guest















          Required, but never shown

























          1 Answer
          1






          active

          oldest

          votes








          1 Answer
          1






          active

          oldest

          votes









          active

          oldest

          votes






          active

          oldest

          votes









          0












          $begingroup$

          The inclusion $mathcal F'subset mathcal F_T$ may be strict.



          Example: Let $(Omega,mathcal F,Bbb P)$ be a probability space on which is defined a standard normal random variable $Z$. (So that $Bbb P[Z=0]=0$.) Let $(mathcal F_t)$ be the filtration generated by the process $X$ defined by
          $$
          X_t=cases{1,&$0le t<1$,cr Z,&$tge 1$.cr}
          $$

          For $T_n$ take the constant $1-1/n$, and then $T=1$. Yiu have $mathcal F_{T_n}=mathcal F' = {emptyset,Omega}$ for all $n$, but $mathcal F_T=sigma{Z}$.
          The key here is that $T$ is predictable, and "something new" happens at time $T$.






          share|cite|improve this answer









          $endgroup$













          • $begingroup$
            I agree with you that the inclusion may be strict, but your example doesn't work because the OP is assuming that $mathcal{F}_t = mathcal{F}_{t-}$ for all $t$.
            $endgroup$
            – saz
            Jan 20 at 17:56










          • $begingroup$
            Missed that. To make an example with random $T$, start a Brownian motion at 0 and run it until the time $T$ that it first hits 1. At that time have the process jump to a random position independent of the past, with a standard normal law, and then stay there forever.
            $endgroup$
            – John Dawkins
            Jan 21 at 14:57
















          0












          $begingroup$

          The inclusion $mathcal F'subset mathcal F_T$ may be strict.



          Example: Let $(Omega,mathcal F,Bbb P)$ be a probability space on which is defined a standard normal random variable $Z$. (So that $Bbb P[Z=0]=0$.) Let $(mathcal F_t)$ be the filtration generated by the process $X$ defined by
          $$
          X_t=cases{1,&$0le t<1$,cr Z,&$tge 1$.cr}
          $$

          For $T_n$ take the constant $1-1/n$, and then $T=1$. Yiu have $mathcal F_{T_n}=mathcal F' = {emptyset,Omega}$ for all $n$, but $mathcal F_T=sigma{Z}$.
          The key here is that $T$ is predictable, and "something new" happens at time $T$.






          share|cite|improve this answer









          $endgroup$













          • $begingroup$
            I agree with you that the inclusion may be strict, but your example doesn't work because the OP is assuming that $mathcal{F}_t = mathcal{F}_{t-}$ for all $t$.
            $endgroup$
            – saz
            Jan 20 at 17:56










          • $begingroup$
            Missed that. To make an example with random $T$, start a Brownian motion at 0 and run it until the time $T$ that it first hits 1. At that time have the process jump to a random position independent of the past, with a standard normal law, and then stay there forever.
            $endgroup$
            – John Dawkins
            Jan 21 at 14:57














          0












          0








          0





          $begingroup$

          The inclusion $mathcal F'subset mathcal F_T$ may be strict.



          Example: Let $(Omega,mathcal F,Bbb P)$ be a probability space on which is defined a standard normal random variable $Z$. (So that $Bbb P[Z=0]=0$.) Let $(mathcal F_t)$ be the filtration generated by the process $X$ defined by
          $$
          X_t=cases{1,&$0le t<1$,cr Z,&$tge 1$.cr}
          $$

          For $T_n$ take the constant $1-1/n$, and then $T=1$. Yiu have $mathcal F_{T_n}=mathcal F' = {emptyset,Omega}$ for all $n$, but $mathcal F_T=sigma{Z}$.
          The key here is that $T$ is predictable, and "something new" happens at time $T$.






          share|cite|improve this answer









          $endgroup$



          The inclusion $mathcal F'subset mathcal F_T$ may be strict.



          Example: Let $(Omega,mathcal F,Bbb P)$ be a probability space on which is defined a standard normal random variable $Z$. (So that $Bbb P[Z=0]=0$.) Let $(mathcal F_t)$ be the filtration generated by the process $X$ defined by
          $$
          X_t=cases{1,&$0le t<1$,cr Z,&$tge 1$.cr}
          $$

          For $T_n$ take the constant $1-1/n$, and then $T=1$. Yiu have $mathcal F_{T_n}=mathcal F' = {emptyset,Omega}$ for all $n$, but $mathcal F_T=sigma{Z}$.
          The key here is that $T$ is predictable, and "something new" happens at time $T$.







          share|cite|improve this answer












          share|cite|improve this answer



          share|cite|improve this answer










          answered Jan 20 at 17:28









          John DawkinsJohn Dawkins

          13.1k11017




          13.1k11017












          • $begingroup$
            I agree with you that the inclusion may be strict, but your example doesn't work because the OP is assuming that $mathcal{F}_t = mathcal{F}_{t-}$ for all $t$.
            $endgroup$
            – saz
            Jan 20 at 17:56










          • $begingroup$
            Missed that. To make an example with random $T$, start a Brownian motion at 0 and run it until the time $T$ that it first hits 1. At that time have the process jump to a random position independent of the past, with a standard normal law, and then stay there forever.
            $endgroup$
            – John Dawkins
            Jan 21 at 14:57


















          • $begingroup$
            I agree with you that the inclusion may be strict, but your example doesn't work because the OP is assuming that $mathcal{F}_t = mathcal{F}_{t-}$ for all $t$.
            $endgroup$
            – saz
            Jan 20 at 17:56










          • $begingroup$
            Missed that. To make an example with random $T$, start a Brownian motion at 0 and run it until the time $T$ that it first hits 1. At that time have the process jump to a random position independent of the past, with a standard normal law, and then stay there forever.
            $endgroup$
            – John Dawkins
            Jan 21 at 14:57
















          $begingroup$
          I agree with you that the inclusion may be strict, but your example doesn't work because the OP is assuming that $mathcal{F}_t = mathcal{F}_{t-}$ for all $t$.
          $endgroup$
          – saz
          Jan 20 at 17:56




          $begingroup$
          I agree with you that the inclusion may be strict, but your example doesn't work because the OP is assuming that $mathcal{F}_t = mathcal{F}_{t-}$ for all $t$.
          $endgroup$
          – saz
          Jan 20 at 17:56












          $begingroup$
          Missed that. To make an example with random $T$, start a Brownian motion at 0 and run it until the time $T$ that it first hits 1. At that time have the process jump to a random position independent of the past, with a standard normal law, and then stay there forever.
          $endgroup$
          – John Dawkins
          Jan 21 at 14:57




          $begingroup$
          Missed that. To make an example with random $T$, start a Brownian motion at 0 and run it until the time $T$ that it first hits 1. At that time have the process jump to a random position independent of the past, with a standard normal law, and then stay there forever.
          $endgroup$
          – John Dawkins
          Jan 21 at 14:57


















          draft saved

          draft discarded




















































          Thanks for contributing an answer to Mathematics Stack Exchange!


          • Please be sure to answer the question. Provide details and share your research!

          But avoid



          • Asking for help, clarification, or responding to other answers.

          • Making statements based on opinion; back them up with references or personal experience.


          Use MathJax to format equations. MathJax reference.


          To learn more, see our tips on writing great answers.




          draft saved


          draft discarded














          StackExchange.ready(
          function () {
          StackExchange.openid.initPostLogin('.new-post-login', 'https%3a%2f%2fmath.stackexchange.com%2fquestions%2f3080350%2fconvergence-of-sigma-algebra-for-converging-stopping-time%23new-answer', 'question_page');
          }
          );

          Post as a guest















          Required, but never shown





















































          Required, but never shown














          Required, but never shown












          Required, but never shown







          Required, but never shown

































          Required, but never shown














          Required, but never shown












          Required, but never shown







          Required, but never shown







          Popular posts from this blog

          Mario Kart Wii

          What does “Dominus providebit” mean?

          Antonio Litta Visconti Arese